Matematik

Vis at den harmoniske række er divergent

10. november 2022 af azulodukovic - Niveau: Universitet/Videregående

Vis, at for alle k = 1,2,3,... er

\sum_{n=2^{k}+1}^{2^{k+1}} \frac{1}{n} \geq (2^{k+1}- 2^{k})\frac{1}{2^{k+1}} = \frac{1}{2}.

Brug dette til at vise, at den harmoniske række er divergent.

------------------------------------------------------------------------------------------------------------------------------------------------

Jeg kan ikke se hvordan jeg skal begynde på at vise uligheden, og så derfra at den harmoniske række er divergent..


Brugbart svar (0)

Svar #1
10. november 2022 af Soeffi

#0. Der gælder: k = 1, 2, 3...

For fastholdt k får man:

\\ \sum_{n=2^{k}+1}^{2^{k+1}} \frac{1}{n}=\frac{1}{2^{k}+1}+\frac{1}{2^{k}+2}+...+\frac{1}{2^{k}+2^{k}}\geq

\\ \frac{1}{2^{k}+2^{k}}+\frac{1}{2^{k}+2^{k}}+...+\frac{1}{2^{k}+2^{k}}=\frac{2^{k}}{2^{k}+2^{k}}=\frac{1}{2}

Når man lader k gå fra 1 til uendelig, så får man:

\\ \sum_{n=1}^{\infty} \frac{1}{n}= \sum_{n=2^{k}+1}^{2^{k+1}} \frac{1}{n}\;(k=1,2,3...)= \sum_{n=2^{1}+1}^{2^{1+1}} \frac{1}{n}+\sum_{n=2^{2}+1}^{2^{2+1}} \frac{1}{n}+\sum_{n=2^{3}+1}^{2^{3+1}} \frac{1}{n}+...=

\sum_{n=3}^{4} \frac{1}{n}+\sum_{n=5}^{8} \frac{1}{n}+\sum_{n=9}^{16} \frac{1}{n}+...\geq \frac{1}{2}+\frac{1}{2}+\frac{1}{2}+...=\sum_{n=1}^{\infty} \frac{1}{2},\;divergerer


Brugbart svar (0)

Svar #2
10. november 2022 af Soeffi

#1. Rettelse: Det skal nok være...

\\ \sum_{n=1}^{\infty} \frac{1}{n}= \sum_{n=2^{k}+1}^{2^{k+1}} \frac{1}{n}\;(k={\color{Red} 0},1,2,3...)= \sum_{n=2^{0}+1}^{2^{0+1}} \frac{1}{n}+\sum_{n=2^{1}+1}^{2^{1+1}} \frac{1}{n}+\sum_{n=2^{2}+1}^{2^{2+1}} \frac{1}{n}+...=

\sum_{n=1}^{2} \frac{1}{n}+\sum_{n=3}^{4} \frac{1}{n}+\sum_{n=5}^{8} \frac{1}{n}+...\geq \frac{1}{2}+\frac{1}{2}+\frac{1}{2}+...=\sum_{n=1}^{\infty} \frac{1}{2},\;divergerer


Brugbart svar (0)

Svar #3
10. november 2022 af Soeffi

#2. Der er noget galt: Der står...

\\ \sum_{n=1}^{\infty} \frac{1}{n} \geq \sum_{n=1}^{\infty} \frac{1}{2}

...det kan ikke være rigtigt!?


Brugbart svar (0)

Svar #4
10. november 2022 af jl9

#3 Det er vel så rigtig nok, at for alle j=1,2,3...:

\sum_{n=1}^{2^j} \frac{1}{n} \geq \sum_{n=1}^{j} \frac{1}{2}


Skriv et svar til: Vis at den harmoniske række er divergent

Du skal være logget ind, for at skrive et svar til dette spørgsmål. Klik her for at logge ind.
Har du ikke en bruger på Studieportalen.dk? Klik her for at oprette en bruger.